Đến nội dung

minhdat881439 nội dung

Có 580 mục bởi minhdat881439 (Tìm giới hạn từ 16-05-2020)



Sắp theo                Sắp xếp  

#392921 $\left\{\begin{matrix} x^{4}+y^...

Đã gửi bởi minhdat881439 on 03-02-2013 - 20:06 trong Phương trình - hệ phương trình - bất phương trình

Giải hệ phương trình
$\left\{\begin{matrix}
x^{4}+y^{2}=\frac{698}{81}\\
x^{2}+y^{2}+xy-3x-4y+4=0
\end{matrix}\right.$

đây
p\s mod off topic nha



#391088 $\left\{\begin{matrix}x^3y-y^4=7 &...

Đã gửi bởi minhdat881439 on 28-01-2013 - 15:15 trong Phương trình - hệ phương trình - bất phương trình

$\left\{\begin{matrix}x^3y-y^4=7 & & \\ x^2y+2xy^2+y^3=9 & & \end{matrix}\right.$

đây bạn



#383352 Giải phương trình lượng giác $$\sin x\sin 2x\sin 3x...

Đã gửi bởi minhdat881439 on 03-01-2013 - 18:39 trong Phương trình, Hệ phương trình Lượng giác

Giải phương trình
$$\sin x\sin 2x\sin 3x + \cos x\cos 2x\cos 3x = \dfrac{1}{2}$$
Xin cảm ơn nhiều.

PT:$$\Leftrightarrow \frac{1}{2}\cos2x(\cos4x+\cos2x)-\frac{1}{2}\sin2x(\cos4x-\cos2x)=\frac{1}{2}$$
$$\Leftrightarrow \cos2x.\cos4x+1-sin^22x-\cos4x.\sin2x+\sin2x.\cos2x-1=0$$
$$\Leftrightarrow \cos4x(\cos2x-\sin2x)+sin2x(cos2x-sin2x)=0$$
$$\Leftrightarrow (\cos2x-\sin2x)(\cos4x+\sin2x)=0$$
$$\Leftrightarrow \begin{bmatrix}
\cos2x-\sin2x=0(1) & \\
\cos4x+\sin2x=0 (2)&
\end{bmatrix}$$
(1):$$\Rightarrow x=\frac{\Pi }{4}+k\Pi $$

(2):$$\Rightarrow\begin{bmatrix}
x=\frac{-\Pi }{12}+k\Pi & \\
x=\frac{7\Pi }{12}+k\Pi & \\
x=\frac{\Pi }{4}+k\Pi &
\end{bmatrix}$$
Vậy phương trình có nghiệm: $x=\frac{-\Pi }{12}+k\Pi;x=\frac{7\Pi }{12}+k\Pi;x=\frac{\Pi }{4}+k\Pi(k\in\mathbb{Z})$



#382190 $\left\{\begin{matrix} x^{3}y-y^...

Đã gửi bởi minhdat881439 on 31-12-2012 - 09:52 trong Phương trình - hệ phương trình - bất phương trình

Giải hệ phương trình
$\left\{\begin{matrix}
x^{3}y-y^{4}=7\\x^{2}y+2xy^{2}+y^{3}=9
\end{matrix}\right.$

Cách khác:

HPT $\Leftrightarrow \left\{\begin{matrix} y(x^{3}-y^{3})=7 & \\ y(x+y)^{2}=9\Rightarrow x> y> 0 & \end{matrix}\right.$
$\Rightarrow x=\frac{3}{\sqrt{y}}-y$ thay vào phương trình đầu ta được:
$y[(\frac{3\sqrt[4]{8}}{ \sqrt{y} }-y)^{3}-y^{3}]=7$
Đặt $t=\sqrt{y}> 0$ thì:
$t^{2}[(\frac{3}{t}-t^{2})^{3}-t^{6}]=7 \Leftrightarrow t^{9}-(3-t^{3})^{3}+7t=0$
Xét hàm số f(t)=$t^{9}-(3-t^{3})^{3}+7t=0$
Ta có:f'(t)=$9t^{8}+9t^{2}(3-t^{3})^{2}+7> 0 ;\forall t> 0$
Vậy hàm số f(t) đồng biến trên khoảng $(0;+\infty )$nên nghiệm của hệ phương trình là duy nhất.Dễ thấy hệ có nghiệm $(2;1)$
Vậy....

p\s anh Trong siêng gõ quá :luoi:



#378821 Tuyển tập một số bài phương trình, hệ phương trình thi HSG tỉnh

Đã gửi bởi minhdat881439 on 19-12-2012 - 14:24 trong Phương trình - hệ phương trình - bất phương trình

Hình như ta có thể chứng minh bất đẳng thức: $\frac{1}{\sqrt{1+2x^{2}}}+\frac{1}{\sqrt{1+2y^{2}}}\geq \frac{2}{\sqrt{1+2xy}}$ Với $0\leq x;y\leq \frac{1}{2}$.(Vừa thấy cái này hôm trước nhưng chưa chứng minh được :wacko: ,ai thạo bất đẳng thức chứng minh giùm em với)

đây
----------
P\s:Làm gì có chuyện hình như :icon6: :icon6:



#378681 Hội những người độc thân thích chém gió !

Đã gửi bởi minhdat881439 on 18-12-2012 - 21:21 trong Góc giao lưu

Tình hình là ad nhóm trên FB do ko chịu được sự thật khi mọi người nhận xét về thú cưng của mình nên đã lạm dụng quyền hạn trục xuất 2 thành viên 1 cách vô tội vạ vì tội....dám nói thật :P

Chí lí em cũng đã ra đảo rôi. ĐẢ ĐẢO :icon6: :icon6:

đảo muôn năm :luoi: :luoi:



#378679 $\sqrt{\frac{6}{2-x}}+\sqrt...

Đã gửi bởi minhdat881439 on 18-12-2012 - 21:12 trong Phương trình - hệ phương trình - bất phương trình

Giải phương trình $\sqrt{\frac{6}{2-x}}+\sqrt{\frac{10}{3-x}}=4$


Điều kiện: x<2
Với: $2>x>\frac{1}{2}$
Ta có: $\left\{\begin{matrix}\sqrt{\frac{6}{2-x}}> 2\\ \sqrt\frac{10}{3-x}>2\\ \end{matrix}\right.$
$\Rightarrow \sqrt{\frac{6}{2-x}}+ \sqrt\frac{10}{3-x}>4$(vô lí)
Suy ra $2>x>\frac{1}{2}$ loại (1)
Với $x<\frac{1}{2}$
Ta có: $\left\{\begin{matrix}\sqrt{\frac{6}{2-x}}< 2\\ \sqrt\frac{10}{3-x}<2\\ \end{matrix}\right.$
$\Rightarrow \sqrt{\frac{6}{2-x}}+ \sqrt\frac{10}{3-x}<4\\$ (vô lí)
Suy ra $x<\frac{1}{2}$ loại (2)
Từ (1) và (2) suy ra $x=\frac{1}{2}$



#378664 Hội những người độc thân thích chém gió !

Đã gửi bởi minhdat881439 on 18-12-2012 - 20:40 trong Góc giao lưu

e hèm, ku Đạt có pồ thế mà giấu a nha, Linh e có hình pồ của Đạt hum up lên làm bằng chứng trước toà :P

Mi dám à Linh :angry:



#378377 Hội những người độc thân thích chém gió !

Đã gửi bởi minhdat881439 on 17-12-2012 - 20:31 trong Góc giao lưu

Em đã bảo là cho thằng Đạt out rồi mà :P
Hôm trước nghe con cựu lớp phó nói sau ni họp lớp, lớp mình sẽ có 2 cặp cưới nhau, trong đó có Minh Đạt + Hương Giang :">
@Đạt: chối chi nữa :"> bạn Giang xinh, hiền lại siêng năng cần cù, lại không cao hơn mi =))
....
Mình mới lập nhóm cho hội trên fb :P ai có fb thì kết bạn với mình tại đây
hoặc tham gia vào nhóm Hội những người độc thân thích chém gió VMF

Coi chừng lên lớp t xử nghe Linh Hình đã gửi



#377004 Hội những người độc thân thích chém gió !

Đã gửi bởi minhdat881439 on 12-12-2012 - 11:28 trong Góc giao lưu

@ Đạt >:)

Hình đã gửi



#376557 Hội những người độc thân thích chém gió !

Đã gửi bởi minhdat881439 on 10-12-2012 - 16:37 trong Góc giao lưu

p/s: thằng Đạt ngày mô cụng tới lớp với bạn Hương Giang trong lớp, đề nghị cho out khỏi hội :namtay

Ê chém quá đấy linh =))



#376347 Giải phương trình: $(x+3)^4+(x+5)^4=16$

Đã gửi bởi minhdat881439 on 09-12-2012 - 19:04 trong Phương trình - hệ phương trình - bất phương trình

Giải phương trình:

$(x+3)^4+(x+5)^4=16$

Đối với phương trình có dạng này : $\left ( x-a \right )^4+\left ( x-b \right )^4=c$ $\left ( c\geq 0 \right )$ ,ta đặt $t=\frac{a+b}{2}$ từ đó thế vào phương trình và giải như bình thường, đây là cách giải tổng quát cho dạng bài này



#376195 Phương trình, Hệ phương trình, Bất phương trình qua các đề thi thử năm 2013

Đã gửi bởi minhdat881439 on 09-12-2012 - 09:11 trong Phương trình - hệ phương trình - bất phương trình

bài 33: $\left\{\begin{matrix}x^{4}-2x=y^{4}-y & \\ (x^{2}-y^{2})^{3}=3 & \end{matrix}\right.$

(Đề thi thử lần 1 chuyên Hùng Vương-Phú Thọ)


Đặt: $x + y = a;x - y = b;3 = {c^3}$
Từ phương trình thứ 2 ta có: $ab = c$
$\left\{ \begin{array}{l}
x = \frac{{a + b}}{2} \\
y = \frac{{a - b}}{2} \\
\end{array} \right. \Rightarrow {x^4} - {y^4} = \left( {x + y} \right)\left( {x - y} \right)\left( {{x^2} + {y^2}} \right) = ab\left[ {{{\left( {\frac{{a + b}}{2}} \right)}^2} + {{\left( {\frac{{a - b}}{2}} \right)}^2}} \right] = \frac{{ab}}{2}\left( {{a^2} + {b^2}} \right)$
$2x - y = \left( {a + b} \right) - \frac{{a - b}}{2} = \frac{{a + 3b}}{2} = \frac{{a + {c^3}b}}{2}$
Phương trình thứ nhất trở thành:
$\frac{{ab}}{2}\left( {{a^2} + {b^2}} \right) = \frac{{a + {c^3}b}}{2} \Leftrightarrow c\left( {{a^2} + {b^2}} \right) = a + {c^3}b$
Chúng ta có hệ mới:
\[\left\{ \begin{array}{l}
c\left( {{a^2} + {b^2}} \right) = a + {c^3}b \\
ab = c \\
\end{array} \right.\]
$\begin{array}{l}
\Rightarrow c\left( {{a^2} + \frac{{{c^2}}}{{{a^2}}}} \right) = a + {c^3}b \\
\Leftrightarrow c{a^4} + {c^3} = {a^3} + a{c^4} \Leftrightarrow \left( {ca - 1} \right)\left( {{a^3} - {c^3}} \right) = 0 \\
\Leftrightarrow \left[ \begin{array}{l}
a = c \\
a = \frac{1}{c} \\
\end{array} \right. \\
\end{array}$
Từ đó ta có nghiệm $[\left( {x,y} \right) = \left( {\frac{{\sqrt[3]{3} + 1}}{2},\frac{{\sqrt[3]{3} - 1}}{2}} \right),\left( {\frac{2}{{\sqrt[3]{3}}}, - \frac{1}{{\sqrt[3]{3}}}} \right)]$




#375031 Phương trình, Hệ phương trình, Bất phương trình qua các đề thi thử năm 2013

Đã gửi bởi minhdat881439 on 04-12-2012 - 12:41 trong Phương trình - hệ phương trình - bất phương trình

Bài 30: Giải hệ phương trình: $$\left\{ \begin{array}{c}\sqrt{2x+y+5}-\sqrt{3-x-y}=x^3-3x^2-10y+6(1)\\x^3-6x^2+13x=y^3+y+10(2)\end{array} \right.$$ THPT Thuận Thành số 1 - Bắc Ninh - Lần 1

Pt (2) $\Leftrightarrow (x-2)^3+(x-2)+10=y^3+y+10$
Xét hàm số $f(t)=t^3+t+10$ có $f'(t)=3t^2+1 >0 $ $\forall t$
$\Rightarrow$ hàm số đồng biến trên $\mathbb{R}$ :
Mà $f(x-2)=f(y)\Leftrightarrow y=x-2$ thay vào pt (1) ta được:
$x^3-3x^2-10x+26=\sqrt{3x+3}-\sqrt{5-2x}$ ĐK: $-1\leq x\leq \frac{5}{2}$
$\Leftrightarrow (x+3)(x-2)(x-4)=\frac{3(x-2)}{\sqrt{3x+3}+3}-\frac{-2(x-2)}{\sqrt{5-2x}+1}$
$\Leftrightarrow (x-2)[(x+3)(x-4)-\frac{3}{\sqrt{3x+3}+3}-\frac{2}{\sqrt{5-2x}+1}]=0$
$\Rightarrow x=2$ (thỏa) $\Rightarrow y=0$
Xét:
$(x+3)(x-4)-\frac{3}{\sqrt{3x+3}+3}-\frac{2}{\sqrt{5-2x}+1}=0$
$\Leftrightarrow x^2-x-12=\frac{3}{\sqrt{3x+3}+3}+\frac{2}{\sqrt{5-2x}+1}$
mà $\left\{\begin{matrix} VP>0 & \\ VT< 0 \forall x \in [-1;\frac{5}{2}] & \end{matrix}\right.$ nên pt vô nghiệm
Vậy hệ có nghiệm duy nhất (2;0)



#374887 GPT: $\sqrt{-x^2+4x+5}=x^2-2(1-\frac{6\sqr...

Đã gửi bởi minhdat881439 on 03-12-2012 - 21:06 trong Phương trình - hệ phương trình - bất phương trình

GPT:
$\sqrt{-x^2+4x+5}=x^2-2(1-\frac{6\sqrt{5}}{5})x+\frac{36-9\sqrt{5}}{5}$
p\s ai del phải gửi lại :(
_________________________________________________________
Chính mình đã hỏi bạn là cần del bài nào rồi mà ! Bạn còn kêu ca gì ?



#371002 Xét dãy số ($x_n$):$\left\{\begin{mat...

Đã gửi bởi minhdat881439 on 20-11-2012 - 19:02 trong Dãy số - Giới hạn

Cho a là số thực dương,xét dãy số ($x_n$):
$\left\{\begin{matrix}
x_1=a & \\
x_{n+1}\geq n+2x_n-\sum_{k=1}^{n-1}kx_k & \forall n\geq 1
\end{matrix}\right.$

Tìm $limx_n$



#370996 Chứng minh rằng có vô hạn các số có dạng $a_n=2^n-3$ ($n...

Đã gửi bởi minhdat881439 on 20-11-2012 - 18:50 trong Số học

Chứng minh rằng có vô hạn các số có dạng $a_n=2^n-3$ ($n \geq 2$) đôi một nguyên tố cùng nhau



#370969 Tuyển tập một số bài phương trình, hệ phương trình thi HSG tỉnh

Đã gửi bởi minhdat881439 on 20-11-2012 - 17:04 trong Phương trình - hệ phương trình - bất phương trình

ĐỀ thi HSG tỉnh Vĩnh Phúc (02/11/2012 môn toán- thpt Chuyên)


$\left\{\begin{matrix}
x^{2}+3x+2=\frac{8}{y}-\sqrt{5y-1} & \\
y^{2}+3y+2=\frac{8}{z}-\sqrt{5z-1} & \\
z^{2}+3z+2=\frac{8}{x}-\sqrt{5x-1}&
\end{matrix}\right.$

mod: công thức kẹp trong cặp thẻ đô la ($ ) nhé bạn


Giả sử : $x\geq y\geq z$
Xét $f(t)=t^2+3t+2$ và $g(t)=\frac{8}{t}-\sqrt{5t-1}$ với $t \in [\frac{1}{5};+\infty )$
f(t) là hàm đồng biến,g(t) là hàm nghịch biến trên khoảng $t \in [\frac{1}{5};+\infty )$
Suy ra $f(x)\geq f(y)\Leftrightarrow g(y)\geq g(z)$
Lại có $g(y)\leq g(z)$
Nên $g(y)=g(z)$ suy ra $y=z$
Chứng minh tương tự ta được: $x=y$
Suy ra $x=y=z$
Thế vào phương trình trên ta giải được
$(x;y;z)=(1;1;1)$



#370961 $\left\{\begin{matrix}(\frac{x...

Đã gửi bởi minhdat881439 on 20-11-2012 - 16:48 trong Phương trình - hệ phương trình - bất phương trình

$\left\{\begin{matrix}(\frac{x}{y}+\frac{y}{x)})(x+y)=15 & \\ (\frac{x^{2}}{y^{2}}+\frac{y^{2}}{x^{2}})(x^{2}+y^{2})=85 & \end{matrix}\right.$
Đây là hệ đối xứng nhưng giải theo cách bt thì rất khó, vì có vướng thêm $\frac{x}{y}$ ??????

Điều kiện :$ x,y \neq 0$
Đặt $x=ty$ hệ đã cho trở thành :
$$\left\{\begin{matrix}y(1+\frac{1}{t})(t+1)=15\\y^2(t^2+\frac{1}{t^2})(t^2+1)=85\end{matrix}\right.$$
Suy ra
$$\frac{15^2}{(t+\frac{1}{t})^2(t+1)^2}=\frac{85}{(t^2+\frac{1}{t^2})(t^2+1)}$$
$$\Leftrightarrow \frac{15^2}{(t^2+1)^2(t+1)^2}=\frac{85}{(t^4+1)(t^2+1)}$$
$$\Leftrightarrow \frac{15^2}{(t^2+1)(t+1)^2}=\frac{85}{t^4+1}$$
$$\Leftrightarrow 14t^4-17t^3-17t^2-17t+14=0$$
Giải theo kiểu đối xứng ta $t=2$ hoặc $t=\frac{1}{2}$
Thế vào giải hệ đối xứng loại 1 ra nghiệm là $(x;y)=(2;4) (4;2)$
(Trich Toan tuoi gia )



#370959 Topic về Phương trình và hệ phương trình không mẫu mực

Đã gửi bởi minhdat881439 on 20-11-2012 - 16:38 trong Phương trình, hệ phương trình và bất phương trình

Góp thêm một bài :
Giải PT :
$\sqrt[3]{1 - x} + \sqrt{x + 2} = 1$

Đặt $\left\{\begin{matrix} \sqrt[3]{1-x}=a & \\ \sqrt{x+2}=b; b\geq 0 & \end{matrix}\right.$
$\Rightarrow \left\{\begin{matrix} 1-x=a^3 & \\ x+2=b^2 & \end{matrix}\right.$
$\Rightarrow a^3+b^2=3$
Ta được hệ phương trình:
$\left\{\begin{matrix} a+b=1 & \\ a^3+b^2=3 & \end{matrix}\right.$
$\Leftrightarrow \left\{\begin{matrix} b=1-a & \\ a^3+a^2-2a-2=0 & \end{matrix}\right.$
$\Leftrightarrow \left\{\begin{matrix} b=1-a & \\ (a+1)(a^2-2)=0 & \end{matrix}\right.$
Tới đây dễ rồi



#370945 $\left\{\begin{matrix} \dfrac{{x+2xy}}{{\sqrt[3...

Đã gửi bởi minhdat881439 on 20-11-2012 - 16:12 trong Phương trình - hệ phương trình - bất phương trình

Giải HPT:
$\left\{\begin{matrix} \dfrac{{x + 2xy}}{{\sqrt[3]{x^2 - 2x + 9}}} = x^2 + y \\ \dfrac{{y + 2xy}}{{\sqrt[3]{y^2 - 2y + 9}}} = y^2 + x \end{matrix}\right.$.

Cảm ơn mọi người trước.

$(1)+(2)\Rightarrow \frac{2xy}{\sqrt[3]{x^2-2x+9}}+\frac{2xy}{\sqrt[3]{x^2-2x+9}}=x^2+y^2$

Có $\sqrt[3]{x^2-2x+9}=\sqrt[3]{(x-1)^2+8}\geq2\Rightarrow \frac{2xy}{\sqrt[3]{x^2-2x+9}} \leq xy \Rightarrow VT \leq 2xy$

Mặt khác $x^2+y^2\geq 2|xy|\geq 2xy$.
Tới đây chắc được rồi bạn làm tiếp nha :biggrin: :biggrin:
p\s bạn sửa lại cái tiêu đề đi kìa nếu dài quá bạn có thể lấy cái phương trình đầu rồi ... cũng được ^_^ ^_^



#370849 Cmr:$\sum \overrightarrow{OA}=\overrightarrow...

Đã gửi bởi minhdat881439 on 20-11-2012 - 10:39 trong Hình học phẳng

Cho ngũ giác đều ABCDE nội tiếp đường tròn tâm O.Cmr:$\overrightarrow{OA}+\overrightarrow{OB}+\overrightarrow{OC}+\overrightarrow{OD}+\overrightarrow{OE}=\overrightarrow{0}$

Bài này ở đây rồi
p\s đã có hẳn một topic của Quân về vecto roi bạn gửi thì nên gửi vào đây



#370369 $\left\{\begin{array}{l}x^3+y^3-...

Đã gửi bởi minhdat881439 on 18-11-2012 - 15:51 trong Phương trình - hệ phương trình - bất phương trình

Giải hệ$\left\{\begin{array}{l}x^3+y^3-xy^2=1\\4x^4+y^4=4x+y\end{array}\right.$

Từ pt1 ta có $x^3+y^3-xy^2=1$
$\Rightarrow 4x^4+y^4=(4x+y)(x^3+y^3-xy^2)$
$\Rightarrow 4x^4+y^4=4x^4+y^4+3xy^3-4x^2y^2+x^3y$
$\Rightarrow 3xy^3-4x^2y^2+x^3y=0\Rightarrow xy(x-3y)(x-y)=0$
Tới đây dễ dàng rồi bạn làm tiếp nha



#370365 Giải hệ phương trình với $(1 + x)(1 + {x^2})(1 + {x^4...

Đã gửi bởi minhdat881439 on 18-11-2012 - 15:44 trong Phương trình - hệ phương trình - bất phương trình

Giải hệ phương trình \[\left\{ \begin{array}{l}
(1 + x)(1 + {x^2})(1 + {x^4}) = 1 + y^7 (1)\\
(1 + y)(1 + {y^2})(1 + {y^4}) = 1+x^7 (2)
\end{array} \right.\]

Nhận xét:dễ thấy x=y=0 hoặc x=y=-1
là nghiệm của hệ phương trình vì vậy ta xét các trường hợp như sau:
+Xét $x> 0$,ta có:
$(1)\Rightarrow (1+x)(1+x^{2})(1+x^{4})=1+x+x^{2}+x^{3}+x^{4}+x^{5}+x^{6}+x^{7}> 1+x^{7}\Rightarrow y> x$ thế vào (2) ta có:
$(2)\Rightarrow 1+y+y^{2}+y^{3}+y^{4}+y^{5}+y^{6}+y^{7}> 1+x+x^{2}+x^{3}+x^{4}+x^{5}+x^{6}+x^{7}> 1+x^{7}\Rightarrow x> y$
Từ đó suy ra hệ vô nghiệm, tương tự $y> 0$ hệ vô nghiệm
+Xét $x< -1\Rightarrow 1+x^{7}< 0\Rightarrow 1+y< 0\Rightarrow y< -1$ mà $1+x+x^{2}+x^{3}+x^{4}+x^{5}+x^{6}+x^{7}> 1+x^{7}\Rightarrow y> x$ tương tự $y< -1,ta có x> y$.Vậy hệ vô nghiệm
+Xét $-1< x< 0$ chứng minh tương tự ta có hệ vô nghiệm
Vạy hệ có nghiệm (0;0),(-1;-1)



#370363 $\sqrt{x-\sqrt{x-1}}+\sqrt{x-...

Đã gửi bởi minhdat881439 on 18-11-2012 - 15:32 trong Phương trình - hệ phương trình - bất phương trình

Bạn ơi cái $f'(x)$ phân thức bên trái chưa chắc đã lớn hơn 0 bạn nhé.

Chắc chắn phải lớn hơn 0 rồi bạn ạ
$\frac{1-\frac{1}{2\sqrt{x-1}}}{2\sqrt{x-\sqrt{x-1}}} \geq 0 \forall x\geq 1$
( vì $\frac{1}{2\sqrt{x-1}}\leq 1 ;\forall x\geq 1$)
$\frac{1+\frac{x}{\sqrt{x^{2}-1}}}{2\sqrt{x+\sqrt{x^{2}-1}}}> 0 \forall x\geq 1$
nên $\Rightarrow f'(x)=\frac{1-\frac{1}{2\sqrt{x-1}}}{2\sqrt{x-\sqrt{x-1}}}+\frac{1+\frac{x}{\sqrt{x^{2}-1}}}{2\sqrt{x+\sqrt{x^{2}-1}}}> 0 \forall x\geq 1$